17. Stochastic Processes II

แชร์
ฝัง
  • เผยแพร่เมื่อ 5 ม.ค. 2015
  • MIT 18.S096 Topics in Mathematics with Applications in Finance, Fall 2013
    View the complete course: ocw.mit.edu/18-S096F13
    Instructor: Choongbum Lee
    This lecture covers stochastic processes, including continuous-time stochastic processes and standard Brownian motion.
    License: Creative Commons BY-NC-SA
    More information at ocw.mit.edu/terms
    More courses at ocw.mit.edu

ความคิดเห็น • 166

  • @RamoSFTT
    @RamoSFTT 7 ปีที่แล้ว +126

    University professors, watch and learn. This is how it should be taught!! Spent a week going through my lecture notes and several book and all it took is this video to make it all stick. Thank you Choongbum Lee and thank you MIT.

    • @Elsa-ut1li
      @Elsa-ut1li 6 ปีที่แล้ว +1

      RamoSFTT feeling the same!

  • @nadekang8198
    @nadekang8198 5 ปีที่แล้ว +237

    Great lecture. When I was in Japan, one of my Japanese professors is also called Ito, and later in his fin. derivatives class, I learned that Ito's Lemma's Kiyoshi Ito was his grandpa.

  • @mhkhusyairi
    @mhkhusyairi 8 ปีที่แล้ว +107

    Clearly the best lecturer in this course

  • @andreapaps
    @andreapaps 3 ปีที่แล้ว +37

    "It looks right but Its wrong for reasons you dont know yet" - Choongbum Lee
    Gonna drop this in my next argument :D

    • @matthewholmes2008
      @matthewholmes2008 3 ปีที่แล้ว +2

      When you're scrolling through the comments on a a random course and you see math guy from wits 10+ years ago :D

    • @andreapaps
      @andreapaps 3 ปีที่แล้ว

      @@matthewholmes2008 Lol dude what were the chances... To be fair it is a maths video :D

    • @johnsamu
      @johnsamu 2 ปีที่แล้ว +2

      "It looks right but Its wrong for reasons you dont know yet".
      That statement is the description of the current state of the worldwide economy.

  • @fastundercoverkitgoogle7381
    @fastundercoverkitgoogle7381 4 ปีที่แล้ว +37

    Choongbum is a legend. The things he is making crystal clear to understand in 1 hour would take you days going over the textbooks and studying the derivations

  • @SeikoVanPaath
    @SeikoVanPaath 3 ปีที่แล้ว +92

    Some notable Timestamps:
    0:00:24 Recap (Lecture 5)
    0:02:14 Continuous-time stochastic process
    0:06:12 (Standard) Brownian Motion
    0:48:08 Quadratic variation theorem

    • @rosea329
      @rosea329 2 ปีที่แล้ว

      🎉🎉🎉🎉🎉😊

    • @ninmarwarda5154
      @ninmarwarda5154 ปีที่แล้ว +1

      you are a legend for giving a very accurate timestamp for all the videos, thanks mate👍

  • @nmecht
    @nmecht 6 ปีที่แล้ว +50

    It's really great when you can see that somebody who clearly knows what they are talking about still takes care to present his knowledge with humility and openness. Great lecturers know enough about their subjects to realize how much they don't know.

    • @cauchyschwarz3295
      @cauchyschwarz3295 2 ปีที่แล้ว +1

      And they respect the results for the hard work they were to derive!

  • @meme_eternity
    @meme_eternity 8 หลายเดือนก่อน +3

    00:00 Stochastic processes and continuous time
    08:48 Brownian motion is a continuous probability distribution
    16:49 Brownian motion is the limit of simple random walks.
    24:26 Brownian motion is a key model used in financial markets and other fields.
    31:45 The probability that the maximum stock price is greater than a at time t is equal to 2 times the probability that the Brownian motion is greater than a.
    41:21 Brownian motion has certain properties
    51:11 Brownian motion has quadratic variation equal to zero
    59:12 Brownian motion and Ito's calculus are used to model stock prices and estimate infinitesimal differences.
    1:07:30 Ito's lemma is a highly nontrivial result that allows for calculus with Brownian motion.
    1:15:38 Calculus using Brownian motion is complicated

  • @wayneqwele8847
    @wayneqwele8847 7 ปีที่แล้ว +22

    What a profoundly simple explanation of that dB^2=dt result.

  • @Anonymous-lw1zy
    @Anonymous-lw1zy 6 ปีที่แล้ว +13

    Wow! What a delightfully clear explanation of Ito's lemma! I love the way Lee steps through his explanations - at each point showing the equation, then illustrating it geometrically, then discussing it in plain English, and then discussing its broader significance. He gives three different ways to understand each step, then ties it all together into the broader mathematical context. Lee is an amazingly gifted teacher!

  • @hongkyukim4179
    @hongkyukim4179 4 ปีที่แล้ว +9

    Dr. Lee is amazing. This is literally the best explanation I've had about Ito's Lemma.

    • @roger_rogers
      @roger_rogers 3 ปีที่แล้ว +1

      hes a quant at millennium now

  • @zonghaoyang6727
    @zonghaoyang6727 4 ปีที่แล้ว +5

    I really like Choonbum's teaching style of how he can break the topic and describe the incentives of each theory. And it is really easier for me to follow as well when professors can write them down on blackboards. I agree now that one can teach math only on blackboard.

  • @e271828r
    @e271828r 5 ปีที่แล้ว +2

    Really enjoyed the lecture. I Made like a bandit sitting on my bed in Jakarta attending this beautiful class in mit Boston many years ago. Thanks ocw and prof choongbum lee

  • @econmajor3996
    @econmajor3996 8 ปีที่แล้ว +2

    Thanks so much, Choongbum.
    Beautiful lecture that delivers important contents, which could have been confusing, in a concise manner. Appreciate your lecture.

  • @Ilija_Ilievski
    @Ilija_Ilievski 7 ปีที่แล้ว

    Absolutely the best teacher I have met so far.

  • @gmandorail
    @gmandorail 8 ปีที่แล้ว +2

    An extraordinary explanation of the fundamentals.

  • @yongliangyang2330
    @yongliangyang2330 4 ปีที่แล้ว +4

    A good lecture that clearly explains the motivation. Even good for engineering people.

  • @mtyjy53
    @mtyjy53 4 ปีที่แล้ว +1

    its even much more useful to study with these few videos about stochastic process than to actually attend lecture at a current university

  • @NgardSC
    @NgardSC 6 ปีที่แล้ว +6

    I hope MIT will upload more lessons from Dr Lee. Very pleasant to listen, you can tell he's a humble and knowledgeable person.

  • @sjx2321
    @sjx2321 6 ปีที่แล้ว +12

    I found the proof of the indifferentiablity of Brownian motion not very rigorous. As the differntiability is defined as: for any given \epsilon > 0, there exists \delta >0, s.t. | [B(t+\delta) - B(t)]/\delta - A | < \epsilon. The inequality derived from the hypothesis that B(t) is differentiable should be | B(t+\delta) - B(t)| < \delta*A + \epsilon. Also notice here I followed the traditional Epsilon-Delta definition here, which means the \epsilon used in the video should be the \delta I used here. Although the instructor's proof captured the intuition and the extra \epsilon term here should not effect the conclusion since \epsilon can be any given fixed positive value, which can be chosen as close to one as possible (I suppose that is what he meant by 'almost smaller or equal to'), I still feel like pointing this out in case anyone has a similar concern, like me.

  • @navidmohammadzadeh5890
    @navidmohammadzadeh5890 6 ปีที่แล้ว

    Clear and Amazing intuition I got from the stochastic process. Appreciate

  • @user-ok4wr4zm5i
    @user-ok4wr4zm5i 2 ปีที่แล้ว

    a combination of clarity and simplicity

  • @byunghunoh76
    @byunghunoh76 5 ปีที่แล้ว +2

    23:26
    I think 'motion of pollen' is better expression than 'action of pollen'.
    In physics, 'action' is a terminology regarding a fundamental dynamic symmetry.

  • @bowencao8656
    @bowencao8656 7 หลายเดือนก่อน

    The explanation of Quadratic Variations is great

  • @Soumirai
    @Soumirai 7 ปีที่แล้ว

    Thank you very much, great course, great explanations!

  • @yunhokim7846
    @yunhokim7846 7 หลายเดือนก่อน

    very clear explanation! so glad this is for free.

  • @lewismambo8043
    @lewismambo8043 4 ปีที่แล้ว +1

    These Lectures are more attractive for those who are working in stochastic analysis.

  • @jannak8824
    @jannak8824 8 ปีที่แล้ว +1

    way too good explained, thanks

  • @68007426
    @68007426 6 ปีที่แล้ว

    This is great! very intuitive and clear.

  • @chuckles8519
    @chuckles8519 5 ปีที่แล้ว

    Excellent lecturer. Smalle quibble with the explanation of Brownian motion as the limit of a simple random walk as the step-size goes to zero: you also need to scale the possible values the process can take at each step in order to recover Brownian motion in this way.

  • @user-qp2ps1bk3b
    @user-qp2ps1bk3b 4 ปีที่แล้ว

    This was a very interesting lecture, thank you!

  • @richardk111
    @richardk111 2 ปีที่แล้ว +1

    This was a great lecture on Brownian motion! Choongbum is truly really talented as a lecturer. It's too bad that he's no longer in academia.

    • @Anyone.c
      @Anyone.c 2 ปีที่แล้ว

      What does he do now?

    • @jasonwang9987
      @jasonwang9987 ปีที่แล้ว

      @@Anyone.c now millennial portfolio manager

  • @xinqiaoli4499
    @xinqiaoli4499 2 ปีที่แล้ว

    This prof is a legend!

  • @zhengyang6066
    @zhengyang6066 3 ปีที่แล้ว +2

    I was so messed up and had no idea what the hell my professor was talking about until I watched this video. Great lecture indeed!

  • @M4TT210
    @M4TT210 8 ปีที่แล้ว +2

    Great Professor! Even his voice is confortable to be listened to.

  • @ufccasero
    @ufccasero 3 ปีที่แล้ว

    AMAZING lecture! Thaks

  • @maxwellsdaemon7
    @maxwellsdaemon7 6 หลายเดือนก่อน

    The more I watch this series of videos on Stochastic processes, the more I realize that while they are good and give some interesting results, the sloppiness of the presentation also shows more and more. Another example is the argument made in 58:35. I thought the presenter should have made a comment about the standard deviation of the Y_i, not just the expectation of the Y_i

  • @user-hi3yc3lq6r
    @user-hi3yc3lq6r ปีที่แล้ว

    Thank you MIT

  • @robertvolgman6590
    @robertvolgman6590 2 ปีที่แล้ว +3

    In the calculations shown at 39:06, why does P(Tau < t) = P(B(t) - B(Tau)>0 | Tau

  • @marcfournier-carrie9481
    @marcfournier-carrie9481 ปีที่แล้ว

    really great lecture

  • @justbecause9613
    @justbecause9613 ปีที่แล้ว

    Helpful lecture. Thank you

  • @waseefullahbsmathematicsst9901
    @waseefullahbsmathematicsst9901 ปีที่แล้ว

    Great lecture

  • @linchenpal
    @linchenpal 4 ปีที่แล้ว +1

    Gracias MIT ❤️

  • @faryalfatima1713
    @faryalfatima1713 6 ปีที่แล้ว +2

    can anyone tell what will be the quadratic variation of an absolute value.or how can we apply ito's formula on skew brownian motion

  • @user-fc1in3mt6g
    @user-fc1in3mt6g 8 ปีที่แล้ว

    very very nice lecture

  • @Boringpenguin
    @Boringpenguin 3 ปีที่แล้ว

    38:32 Just a side note, Dr.Lee's argument is actually hinting on the strong Markov property of Wiener processes.

  • @thedthai1
    @thedthai1 7 ปีที่แล้ว

    Thank you very much

  • @yuewang5008
    @yuewang5008 5 ปีที่แล้ว

    thank you so much

  • @nirmalya15
    @nirmalya15 5 ปีที่แล้ว +1

    excellent

  • @haneulkim4902
    @haneulkim4902 9 หลายเดือนก่อน

    Amazing lecture!
    Could someone clarify my understanding. @11:48, the second point(stationary) in the theorem. Does it mean that if we take many multiple interval from one brownian motion path ~N(0,t-s) or from multiple brownian paths , same interval is taken then it has distribution ~N(0, t-s). → I think it is the latter since first proposition does not make sense, if multiple intervals are taken how to find variance?

  • @annymoose1046
    @annymoose1046 ปีที่แล้ว

    Probability of "1" but no longer isolated intervals if the system is mapped in a finite four-dimensional tesseract where a Brownian particle moves in and out of 3 dimensional space

  • @maxwellsdaemon7
    @maxwellsdaemon7 6 หลายเดือนก่อน

    3:30, the stock price can't be negative, so how can it be modeled by brownian motion (or continuous unbiased random walk)?

  • @user-ok4wr4zm5i
    @user-ok4wr4zm5i 2 ปีที่แล้ว

    The mathematical expectation of the product of independent random variables is equal to the product of their mathematical expectations

  • @liwang248
    @liwang248 5 ปีที่แล้ว +1

    Oh my god. Math becomes beautiful again to me!!! First time to know that Ito lemma comes from tayler expansion after learning ito lemma for a year from my school. Thanks MIT!!!!! I should have accepted your admission. Regret.

  • @minymaker
    @minymaker ปีที่แล้ว

    Tank you so much!!

  • @draymondgao8045
    @draymondgao8045 4 ปีที่แล้ว +1

    Around 16:06. I think it should be Z(t/n) = Y_t / (sqrt(n)). Because if Z is a standard Brownian motion, then Z(1) ~ N(0,1). if Z(t/n) = Y_t, just as what the lecturer told, then Z(1)=Z(n/n)=Y_n ~ N(0,n), not N(0,1). Is it right?

  • @thomasbernhard8489
    @thomasbernhard8489 5 ปีที่แล้ว

    Thanks

  • @trstrscit
    @trstrscit 4 หลายเดือนก่อน

    53.15 min...Quadratic variation square is always possitive..and we are summing to infinity...even if its very less initially as we add up more and more it shud be positive number right? How it will become zero for normal function? It looks like counter intuitive

  • @ruanrichter
    @ruanrichter 7 ปีที่แล้ว +1

    @37:07 I'm slightly confused. The line he writes down then, the third line, is where I don't fully understand. I'ts a simple proof but for some reason I'm struggling here. The third line says; "probability of the process ending up above "a", at time "t", given it hits "a" before time "t"....+ the same for ending up below "a" at time "t"... basically.. if we've already hit "a" before time "t" (given that Tau

    • @ruanrichter
      @ruanrichter 7 ปีที่แล้ว +2

      Never mind :) I see he fixes it afterwards :)

  • @dhnguyen68
    @dhnguyen68 ปีที่แล้ว

    I believe the 3rd name of Brownian motion (aka Wiener process) is Levy process named after a French mathematician Paul Levy

  • @ChampWinJim
    @ChampWinJim ปีที่แล้ว

    Wondering if brownian motion can be controlled with force fields 🤔

  • @dennisestenson7820
    @dennisestenson7820 2 ปีที่แล้ว

    19:44 uhhhm, pollen particles are very tiny and generally float on water. And Brownian motion is best observed at the surface of water under a microscope, where gravity is canceled out by surface tension. However, he's only considering the motion parallel to the surface of the water, so... hopefully someone has informed him of what pollen particles are by now.

  • @corinachen4954
    @corinachen4954 3 ปีที่แล้ว +1

    this is n times better than my lecturer, as n -> \infty

  • @beaugogh4490
    @beaugogh4490 ปีที่แล้ว

    Around 1:07, if Bt was differentiable, why df=(dBt/dt)dt? Doesn't it become df = dBt? I am confused about this part, can anyone explain?

  • @kingshukdutta2064
    @kingshukdutta2064 2 ปีที่แล้ว +1

    Why are the 3rd order and so on terms ignored in Ito's lemma?

  • @linchen4535
    @linchen4535 6 ปีที่แล้ว

    Emmm I got a question for the proof around 39min. What if price goes down after tau_a less that t. Then P(M(t) > a) is zero and P(tau_a < t)is 1. These two should not be equal?

    • @sjx2321
      @sjx2321 6 ปีที่แล้ว

      If the price goes down after \tau_a and never manage to climb back before t (actually, it doesn't really matter whether the price crosses the line y=a again, since you can always find a symmetric path that ends on the other side of y=a, as long as B(t) < a, we are good), that would be the case covered by the second part on the RHS, which is P(B(t)

    • @heartofwater3805
      @heartofwater3805 6 ปีที่แล้ว

      I think there is a "typo" in defining τ_a. But everything will go through if we define τ_a = inf {s ≥ 0 | B(s)>a}.

  • @vladimir0681
    @vladimir0681 4 ปีที่แล้ว +2

    In the proof of the property at 39:00 he uses conditional probability P(B(t)-B(tau_a)>0 | tau_a

  • @tizweber8871
    @tizweber8871 4 ปีที่แล้ว

    53:11 where did we use the continouity of the derivative ?
    Is it when we use the fact ,that a continous function is locally bounded ?

    • @kamille49
      @kamille49 4 ปีที่แล้ว +1

      By use of mean value theorem ?

    • @Boringpenguin
      @Boringpenguin 3 ปีที่แล้ว

      Jude is right, MVT requires the function to be continuous.

  • @francoisallouin1865
    @francoisallouin1865 5 ปีที่แล้ว +5

    arround 1:07:00 the chaine rule is wrong. It is corrected one minute later.

    • @danilobattezzato
      @danilobattezzato 4 หลายเดือนก่อน

      Any idea what the correct statement of the chain rule would be? the video doesn't show the board when he's correcting it

  • @user-bh9ei9fl1z
    @user-bh9ei9fl1z 4 ปีที่แล้ว +1

    I have a question on the third equation at 40:31. Why B(t)=B(\tau_a) not considered?

    • @tomjordan6092
      @tomjordan6092 4 ปีที่แล้ว

      Because it is related to P(B(t)>a) and B(tau_a) is equal to a, so P(B(t))= B(tau_a) is not applicable. It has to be greater or less than a, whilst also crossing a

    • @jivillain
      @jivillain 3 ปีที่แล้ว

      Tom Jordan why is P(M(t) > a) = P(B(t)-B(tao)>0 U tao

    • @Boringpenguin
      @Boringpenguin 3 ปีที่แล้ว +1

      Because it accounts for no mass at all, if X is a continuous R.V., then P(X = x) is always 0
      You can incorporate it to either the 1st or 2nd term, but it makes no difference from a distribution point of view.

  • @chzpan
    @chzpan 6 หลายเดือนก่อน

    1:06:24 This function f(Bt) is not smooth as it's non-differentiable at S0.

  • @user-ok4wr4zm5i
    @user-ok4wr4zm5i 2 ปีที่แล้ว

    it was necessary to note the property of the sum of normal distributions

  • @yifuliu8419
    @yifuliu8419 7 ปีที่แล้ว +7

    Frankly, 58:06 the proof is not very clear, espcially he didn't give the step from X~N(0,T/N) to E(X^2)=T/N. He used the formula Var(X)=E(X^2)-E^2(X) here

    • @kislayaravi24
      @kislayaravi24 6 ปีที่แล้ว +4

      Var(X)=E(X^2)-E^2(X)
      Now E(X) = 0
      This makes Var(X) = E(X^2)
      and Var(X) = T/n
      This makes E(X^2) = T/n

  • @user-ok4wr4zm5i
    @user-ok4wr4zm5i 2 ปีที่แล้ว

    fit to the answer

  • @wcottee
    @wcottee 8 ปีที่แล้ว

    Do we really mean Brownian motion reflects the price? Brownian motion can go negative but prices cannot. Any help appreciated.

    • @torinogranatafair
      @torinogranatafair 8 ปีที่แล้ว +3

      this is why actually it is stock price return and not stock price that is modelled in this way. It is explained in some way at 01:01:00 , dS/St is brownian motion

    • @wcottee
      @wcottee 8 ปีที่แล้ว

      Thanks Federico...will take a look.

    • @robertvolgman6590
      @robertvolgman6590 2 ปีที่แล้ว

      @@wcottee i know these comments are old but F_B is technically incorrect. dS/St is not brownian motion, it is the stochastic equivalent of derivative of brownian motion (i.e. normally distributed with a mean of 0). the answer to wcottee's question is that generally you model the log of stock prices with brownian motion, and the log can go below zero. Since brownian motion processes are invariant under shifts you can also define the process such that the brownian motion starts at the current log of the stock price. When you undo the logarithm, this effectively results in the % returns being normally distributed around 0.

    • @wcottee
      @wcottee 2 ปีที่แล้ว

      @@robertvolgman6590 Thanks for the reply, Robert.

  • @jivillain
    @jivillain 3 ปีที่แล้ว

    Can someone explain why P(M(t) > a) = P(B(t)-B(tao)>0 and tao

    • @Boringpenguin
      @Boringpenguin 3 ปีที่แล้ว

      Let's denote the time range as 0 = a}, but it makes no difference from a distribution point of view)
      If you think about it, B(s) at time t can really only be either above the level a, or below (Again I am ignoring the equality case here)
      You can choose an arbitrary reference level and the statement will still be true. Just so happens choosing a in this case is the most natural thing to do, as demonstrated by the calculations in the video.
      So, these are the only two possibilities no matter what conditional event you impose on them, that's why we can break P(\tau_a < t) into the sum above.

    • @dhruvvansrajrathore2148
      @dhruvvansrajrathore2148 3 ปีที่แล้ว +1

      Let A,B be two sets. Is it easy to see that P(A)=P(A and B)+P(A and B^{c})?
      Then if you set A={t:tao0} does it works?

  • @ninmarwarda5154
    @ninmarwarda5154 ปีที่แล้ว

    At 1:13:15 mark, I still don't understand why are we working out the taylor expansion up until the second derivative only? why not up until 5 or 6 or even infinity?

    • @lazywarrior
      @lazywarrior ปีที่แล้ว

      because the rest of the terms goes to 0 as t approaches 0. See it now?

    • @ninmarwarda5154
      @ninmarwarda5154 ปีที่แล้ว

      @@lazywarrior hmmm and so this happens due to the Quadratic variation concept, right?

    • @lazywarrior
      @lazywarrior ปีที่แล้ว

      @@ninmarwarda5154 no. This is just evaluating the limit. My previous reply contain typos. I should say as delta t approaches 0. Remember that big O notation in some form of Taylor expansion? That remainder = zero at the limit.

  • @MohamedNidabdella
    @MohamedNidabdella 7 ปีที่แล้ว

    Thank you !

  • @sinhnguyen4815
    @sinhnguyen4815 3 ปีที่แล้ว

    This stuff is his right hand stuff eh. It's technical but he seems to know it though. He just has to look it up in a book if he needs it. I think he was playing. Reality has boundaries. It's very small. The rest is just cooking. But it can be studied.

  • @abbassalimi5600
    @abbassalimi5600 6 ปีที่แล้ว

    Can someone please explain why (dB(t))2=dt . Thank you

    • @CraaaabPeople
      @CraaaabPeople 6 ปีที่แล้ว +4

      Because of quadratic variation being non-zero.

    • @MiniNinjaUK1
      @MiniNinjaUK1 5 ปีที่แล้ว

      when he says we take the infinitesimal difference of the LHS, he just ignores the sum. How does one get around this? Thank you

    • @Boringpenguin
      @Boringpenguin 3 ปีที่แล้ว +2

      @@MiniNinjaUK1 Note that we can rewrite the RHS into something like RHS = sum_{i = 1}^{n} [t_{i} - t_{i-1}], this is a telescoping sum which evaluates to T.
      Then very informally speaking, when n is crazily large, we can treat t_{i} - t_{i-1} as super small time increment, which is like dt
      By the same heuristic, we can also treat (B_{t_{i}} - B_{t_{i-1}})^2 as a very small change of B(t) all squared (whatever that is), and we named it (dB(t))^2
      At first, it may be kind of confusing why we even adopted such a naming, since we were using calculus notation for something that is inherently not differentiable everywhere a.s.
      However, if you accept the "abusing of notation" here and move on to the next lecture, you will see why we restate the quadratic variation in this "differential" form and how powerful it is.

  • @maxwellsdaemon7
    @maxwellsdaemon7 6 หลายเดือนก่อน

    34:13, "tau_a = min_t { B(t)=a }" This notation looks wrong.

  • @eb6195
    @eb6195 3 ปีที่แล้ว +1

    I like to see examples in the world to make theoretical more clear. Printing presses operate by printing color dot patterns. You need a magnifying glass to see the dots. In the printing field there is an occasionally used technique of computer generated (yes, an oxymoron) random dot pattern of C, M, Y, K (basically primary colors + black) (as opposed to standard evenly spaced dots) called stochastic dot pattern. It is used for a high end fine art look and to prevent moire patterns.

  • @samuelblattner7733
    @samuelblattner7733 3 ปีที่แล้ว

    Can someone help me out with something, whether I'm think about this correctly? He seems to prove that for each t, the brownian motion is non-differentiable wp1, which is different from the third property he writes on the board that brownian motions are nowhere differentiable wp1. What he proves is that after fixing a t, the collection of paths that are not differentiable at that particular t is measure 1. If this collection of paths were the same for each t, then you'd get a full measure set of paths that are nowhere differentiable. But if for each t you get a different measure zero set of paths that are differentiable at t, since there are uncountably many t, the union of these measure 0 sets is not necessarily measure 0. This would imply that the set of nowhere differentiable paths is not measure one... so it seems like he proved something different from what he wrote. Is that right?

    • @Boringpenguin
      @Boringpenguin 3 ปีที่แล้ว

      I think your confusion partly stems from the fact that he is not actually proving the exact version of the 3rd property. As it is just a heuristic "proof", inevitably there will be some logical flaws/holes hidden within.
      If you have a strong measure-theoretic probability background, then maybe skimming through the original proof made by Dvoretsky, Erdos, and Kakutani would be a good choice.

  • @alexander5095
    @alexander5095 ปีที่แล้ว

    I don't understand the proof for P(M(t) > a). Isn't P( B(t) - B(tau_a) >0 | tau_a

  • @sandspatel
    @sandspatel 4 ปีที่แล้ว

    Just goes to show a good teacher can make complex content a lot more accessible. My professor was awful in comparison.

  • @HHAashish
    @HHAashish 3 ปีที่แล้ว +1

    Ich küss' Dein Auge. Ehrenmann.

  • @mm22sapphire50
    @mm22sapphire50 7 ปีที่แล้ว

    wish i had his mind

  • @chunchuanlv3211
    @chunchuanlv3211 6 ปีที่แล้ว

    Am I the only one who thinks at th-cam.com/video/PPl-7_RL0Ko/w-d-xo.html P(\tau_a

  • @brown_town_
    @brown_town_ 8 ปีที่แล้ว +7

    0:27 black board is clean as shit

    • @om251204
      @om251204 7 ปีที่แล้ว +4

      shit is not the right word to use for such situations.

  • @greyreynyn
    @greyreynyn 7 ปีที่แล้ว +3

    can't tell if these comments are sarcastic or not...

    • @hellokalp5809
      @hellokalp5809 6 ปีที่แล้ว

      greyreynyn why would they be sarcastic?

    • @eb6195
      @eb6195 3 ปีที่แล้ว +1

      Do you mean stochastic?

  • @onurboyar3345
    @onurboyar3345 4 ปีที่แล้ว

    sen adamsin

    • @onurboyar3345
      @onurboyar3345 4 ปีที่แล้ว

      sorry for my sexist language

  • @Elsa-ut1li
    @Elsa-ut1li 6 ปีที่แล้ว

    Is this lecture for math students or financial students?

    • @mitocw
      @mitocw  6 ปีที่แล้ว +4

      From the course description: "The purpose of the class is to expose undergraduate and graduate students to the mathematical concepts and techniques used in the financial industry. Mathematics lectures are mixed with lectures illustrating the corresponding application in the financial industry. MIT mathematicians teach the mathematics part while industry professionals give the lectures on applications in finance." See the course on MIT OpenCourseWare for more information and materials at ocw.mit.edu/18-S096F13.

    • @jivillain
      @jivillain 3 ปีที่แล้ว +1

      For quants

  • @user-ok4wr4zm5i
    @user-ok4wr4zm5i 2 ปีที่แล้ว

    0.5/0.1 =0.5/0.1 =5 but don't be equal 0.5

  • @michalbotor
    @michalbotor 5 ปีที่แล้ว

    i have a vague feeling, that the 37:02 equality: P(M(t)>a) = P(tau_a < t) might not always be true. Namely M(t) > a means, that in the span of time interval (0,t), at some time s, we had B(s) > a, whereas tau_a < t means, that in the span of time interval (0,t), at some time s=tau_a, we had B(tau_a) = B(s) = a. what if in the latter case in the span of time interval (tau_a, t) we had B(s)

    • @ichbinda1966
      @ichbinda1966 5 ปีที่แล้ว

      michał botor why should M(t)

    • @michalbotor
      @michalbotor 5 ปีที่แล้ว

      oh, boy. i just realized, that this is even more complicated then i originally thought, so let me restate the above again, and hopefully this will help you to resolve your doubts as well. unless my reasoning is flawed, of course. ;)
      so..
      M(t) records the maximum value of brownian motion in the time interval (0, t).
      tau_a records *the first time* brownian motion attains the value of a.
      moreover, since (by definition of brownian motion) B(0) = 0 and t -> B(t) is a continuous function, then brownian motion starts from 0 and cannot jump in value, i.e. (1) to reach the value of a it must go through all of the values between 0 and a first, and (2) to reach the value greater than a it must reach the value of a first, which i can summarize as follows if 0< b < a < c, then tau_b < tau_a < tau_c.
      what i'm trying to say is that because of the two mentioned defining properties of brownian motion tau_a not only records for you the moment when brownian motion first attained the value of a, but also implicitly tells you, that in the past it never exceeded the value of a, and so it must be, that M(tau_a) = a.
      now, to answer your question, if we assume that tau_a < t, then from what i said above at time t maximum value of browinan motion M(t) is at least as big as it was at time tau_a, namely M(tau_a) = a, and if it is greater than a, then it must have reached this greater value after time tau_a. however if at time tau_a brownian motion will bounce off of the value of a and never reach it again in the span of time interval (tau_a, t), then its maximum value will be the one that was recored at time tau_a, i.e. M(t) = M(tau_a) = a.
      on the contrary if we assume, that M(t) > a, then well M(t) > a.
      an this is the reason, why i'm worrying, that the equality P(M(t)>a) = P(tau_a

    • @jivillain
      @jivillain 3 ปีที่แล้ว

      michał botor i have this same confusion. Have you figured it out?

    • @Boringpenguin
      @Boringpenguin 3 ปีที่แล้ว +1

      @@michalbotor That's why a more "careful" treatment will be using the set {M(t) >= a} instead of simply ignoring the equality case
      In distributional sense they are both the same, but the argument seems to be more solid if we directly consider P(M(t) >= a)

    • @robertvolgman6590
      @robertvolgman6590 2 ปีที่แล้ว

      @@michalbotor Because the process is continuous the probability that M(t) is any arbitrary constant is 0, so P(M(t) = a)=0, so the statement is still true. You are right that the process could "bounce" off a and never reach it again, but in a brownian motion process this is infinitely unlikely to happen.

  • @user-wk7qp6vc8p
    @user-wk7qp6vc8p ปีที่แล้ว

    I don't think he proves Brownian motion is nowhere differentiable correctly.

  • @user-ok4wr4zm5i
    @user-ok4wr4zm5i 2 ปีที่แล้ว

    Quadratic variation theorem he's confused by the wrong proof

  • @luisescarcega
    @luisescarcega 3 ปีที่แล้ว

    57:50 I cannot believe he made that silly mistake...

    • @jure4835
      @jure4835 2 ปีที่แล้ว

      What mistake?

  • @saintelohim
    @saintelohim 4 ปีที่แล้ว

    1:04:12 f is a smooth function. 1:05:03, f is not smooth at S_0

  • @jeanphillippes2196
    @jeanphillippes2196 4 ปีที่แล้ว

    I didn't come here for a lecture.

  • @user-ok4wr4zm5i
    @user-ok4wr4zm5i 2 ปีที่แล้ว

    f'(s) in square